2001 AIME II Problems/Problem 3

Revision as of 00:40, 20 November 2007 by Minsoens (talk | contribs)

Problem

Given that

$\begin{eqnarray*}x_{1}&=&211,\\ x_{2}&=&375,\\ x_{3}&=&420,\\ x_{4}&=&523, \textrm{ and}\\ x_{n}&=&x_{n-1}-x_{n-2}+x_{n-3}-x_{n-4}\textrm{ when }n\geq5, \end{eqnarray*}$ (Error compiling LaTeX. Unknown error_msg)

find the value of $x_{531}+x_{753}+x_{975}$.

Solution

This problem needs a solution. If you have a solution for it, please help us out by adding it.

See also

2001 AIME II (ProblemsAnswer KeyResources)
Preceded by
Problem 2
Followed by
Problem 4
1 2 3 4 5 6 7 8 9 10 11 12 13 14 15
All AIME Problems and Solutions